Ableitung der Lagrange-Dichte für das elektromagnetische Feld

Wenn ich das (spezielle) relativistische EM-Feld betrachte, verstehe ich das unter der Annahme einer Lagrange-Dichte der Form

L = 1 4 F μ v F μ v + 1 C J μ A μ

und das Befolgen der Euler-Lagrange-Gleichungen stellt die Maxwell-Gleichungen wieder her.

Gibt es eine First-Principle-Ableitung dieser Lagrange-Funktion? Eine Referenz oder Erklärung wäre sehr willkommen!

Von welchen Grundsätzen wollen Sie ausgehen?

Antworten (5)

Abstrakt

Im Folgenden werden wir beweisen, dass es eine kompatible Lagrange-Dichte für das elektromagnetische Feld im leeren Raum gibt

(045) L e M = ϵ 0 | | E | | 2 C 2 | | B | | 2 2 ρ ϕ + J A
das heißt, die Euler-Langrange-Gleichungen, die aus dieser Lagrange-Funktion hervorgehen, sind die Maxwell-Gleichungen für das elektromagnetische Feld.

Diese Lagrange-Dichte wird durch ein Trial-and-Error- Verfahren (1) abgeleitet , nicht durch Raten.

1. Einleitung

Die Maxwellschen Differentialgleichungen des elektromagnetischen Feldes im leeren Raum sind

(001a) × E = B T (001b) × B = μ 0 J + 1 C 2 E T (001c) E = ρ ϵ 0 (001d) B = 0
Wo E = elektrischer Feldstärkevektor, B = magnetischer Flussdichtevektor, ρ = elektrische Ladungsdichte, J = elektrischer Stromdichtevektor. Alle Größen sind Funktionen der drei Raumkoordinaten ( X 1 , X 2 , X 3 ) ( X , j , z ) und Zeit T X 4 .

Aus Gleichung (001d) der Magnetflussvektor B kann als Kräuselung eines Vektorpotentials ausgedrückt werden A

(002) B = × A
und aus (002) ergibt Gleichung (001a).
(003) × ( E + A T ) = 0
daher kann der Klammerterm als Gradient einer Skalarfunktion ausgedrückt werden
E + A T = ϕ
das ist
(004) E = ϕ A T
Also die sechs skalaren Variablen, die Komponenten von Vektoren E Und B , können als Funktionen von 4 Skalarvariablen, dem Skalarpotential, ausgedrückt werden ϕ und drei Komponenten des Vektorpotentials A .

Einfügen der Ausdrücke von E Und B , Gleichungen (002) bzw. (004), in den Gleichungen (001b) und (001c) haben wir

(005) × ( × A ) = μ 0 J + 1 C 2 T ( ϕ A T )

Und
(006) 2 ϕ T ( A ) = ρ ϵ 0
Angesichts dessen
(007) × ( × A ) = ( A ) 2 A
Gleichung (005) ergibt
(008) 1 C 2 2 A T 2 2 A + ( A + 1 C 2 ϕ T ) = μ 0 J

2. Die Euler-Lagrange-Gleichungen des EM-Felds

Unsere Hauptaufgabe besteht nun darin, eine Lagrange-Dichte zu finden L , Funktion der vier ''Feldkoordinaten'' und ihrer Ableitungen 1. Ordnung

(009) L = L ( η ȷ , η ȷ , η ȷ ) ( ȷ = 1 , 2 , 3 , 4 )
so dass die vier skalaren elektromagnetischen Feldgleichungen (006) und (008) aus den Lagrange-Gleichungen abgeleitet werden
(010) T [ L ( η ȷ T ) ] + k = 1 k = 3 X k [ L ( η ȷ X k ) ] L η ȷ = 0 , ( ȷ = 1 , 2 , 3 , 4 )
vereinfacht in der Notation zu
(011) T ( L η ȷ ) + [ L ( η ȷ ) ] L η ȷ = 0 , ( ȷ = 1 , 2 , 3 , 4 )

Hier die Lagrange-Dichte L ist eine Funktion von

  1. die vier ''Feldkoordinaten''

(012.1) η 1 = A 1 ( X 1 , X 2 , X 3 , T ) (012.2) η 2 = A 2 ( X 1 , X 2 , X 3 , T ) (012.3) η 3 = A 3 ( X 1 , X 2 , X 3 , T ) (012.4) η 4 = ϕ ( X 1 , X 2 , X 3 , T )

  1. ihre zeitlichen Ableitungen

(013.1) η 1 η 1 T = A 1 T A 1 (013.2) η 2 η 2 T = A 2 T A 2 (013.3) η 3 η 3 T = A 3 T A 3 (013.4) η 4 η 4 T = ϕ T ϕ

Und

  1. ihre Steigungen

(014) η 1 = A 1 , η 2 = A 2 , η 3 = A 3 , η 4 = ϕ

Wir drücken die Gleichungen (006) und (008) in Formen aus, die den Lagrange-Gleichungen (011) ähnlich sind.

(015) T ( A ) + ( ϕ ) ( ρ ϵ 0 ) = 0
Und
(016) T ( A k T + ϕ X k ) + [ C 2 ( A X k A k ) ] J k ϵ 0 = 0
Die Lagrange-Gleichung (011) für ȷ = 4 , das ist für η 4 = ϕ , Ist
(017) T ( L ϕ ) + [ L ( ϕ ) ] L ϕ = 0

Beim Vergleich der Gleichungen (015) und (017) stellen wir fest, dass die erste aus der zweiten abgeleitet werden könnte, wenn

(018) L ϕ = A , L ( ϕ ) = ϕ , L ϕ = ρ ϵ 0

so dass die Lagrange-Dichte L müssen jeweils die Begriffe enthalten
(019) L a 1 ( A ) ϕ , L a 2 1 2 ϕ 2 , L a 3 ρ ϕ ϵ 0
und damit ihre Summe
(020) L a = L a 1 + L a 2 + L a 3 = ( A ) ϕ + 1 2 ϕ 2 ρ ϕ ϵ 0

Wir nehmen an, dass eine geeignete Lagrange-Dichte L wäre von der Form

(021) L = L a + L β
und da L a Gleichung (015) erzeugt, erwarten wir L β , zu bestimmen, wird Gleichungen (016) erzeugen. Diese Erwartung wäre richtig, wenn die Gleichungen (015) und (016) entkoppelt wären, zB wenn die erste enthält ϕ -Begriffe nur und die zweite A -Bedingungen nur. Aber hier ist das nicht der Fall: L a als enthaltend A -Terme würden an der Erstellung von Gleichungen (016) und darüber hinaus teilnehmen L β an der Erzeugung von Gleichung (015) teilnehmen und möglicherweise gegenseitig die Erzeugung der Gleichungen zerstören würden, wie wir es erwartet hatten. Aber hier folgen wir einem Trial-and-Error-Verfahren, das zur richtigen Antwort führt, wie wir im Folgenden sehen werden.

Nun werden die Lagrange-Gleichungen (011) für ȷ = k = 1 , 2 , 3 , das ist für η k = A k , Sind

(022) T ( L A k ) + [ L ( A k ) ] L A k = 0

Beim Vergleich der Gleichungen (016) und (022) stellen wir fest, dass die erste aus der zweiten abgeleitet werden könnte, wenn

(023) L A k = A k + ϕ X k , L ( A k ) = C 2 ( A X k A k ) , L A k = J k ϵ 0

Aus der ersten der Gleichungen (023) die L β Teil der Lagrange-Dichte L muss die Bedingungen enthalten

(024) 1 2 A k 2 + ϕ X k A k , k = 1 , 2 , 3
und so ihre Summe in Bezug auf k
(025) L β 1 1 2 A ˙ 2 + ϕ A ˙

Aus der 2. der Gleichungen (023) die L β Teil der Lagrange-Dichte L muss die Bedingungen enthalten

(026) 1 2 C 2 [ A X k A k A k 2 ] , k = 1 , 2 , 3
und so ihre Summe in Bezug auf k
(027) L β 2 1 2 C 2 k = 1 k = 3 [ A X k A k A k 2 ]
Aus der 3. Gleichung (023) die L β Teil der Lagrange-Dichte L muss die Bedingungen enthalten
(028) J k A k ϵ 0 , k = 1 , 2 , 3
und so ihre Summe in Bezug auf k
(029) L β 3 J A ϵ 0

Aus den Gleichungen (025), (027) und (029) werden die L β Teil der Lagrange-Dichte L Ist

(030) L β = L β 1 + L β 2 + L β 3 = 1 2 A ˙ 2 + ϕ A ˙ + 1 2 C 2 k = 1 k = 3 [ A X k A k A k 2 ] + J A ϵ 0

Schließlich aus den Ausdrücken (020) und (030) für die Dichten L a , L β die Lagrange-Dichte L = L a + L β Ist

(031) L = L a + L β = ( A ) ϕ + 1 2 ϕ 2 ρ ϕ ϵ 0 + 1 2 A ˙ 2 + ϕ A ˙ + 1 2 C 2 k = 1 k = 3 [ A X k A k A k 2 ] + J A ϵ 0 (das ist eine falsche Lagrange-Dichte)

3. Error-Trial-Enderfolg

Einsetzen dieses Lagrange-Dichteausdrucks in die Lagrange-Gleichung bzgl ϕ , also Gleichung (017), ergibt nicht Gleichung (006), sondern

(032) 2 ϕ T ( 2 A ) = ρ ϵ 0 , ( falsch )
Das Erscheinen eines Extras ( A ) liegt an der Laufzeit ( ϕ A ˙ ) von L β und deshalb ist die durch Gleichung (031) gegebene Lagrange-Dichte nicht angemessen.

Um dieses Problem zu lösen, müssen wir (015), also (006), wie folgt aus einem anderen Blickwinkel betrachten

(033) ( ϕ + A ˙ ) ( ρ ϵ 0 ) = 0

Beim Vergleich der Gleichungen (033) und (017) stellen wir fest, dass die erste aus der zweiten abgeleitet werden könnte, wenn wir anstelle von (018) haben

(034) L ϕ = 0 , L ( ϕ ) = ϕ + A ˙ , L ϕ = ρ ϵ 0

also anstelle von (019) bzw. (020) die Gleichungen
(035) L a 1 ' 0 , L a 2 ' 1 2 ϕ 2 + ϕ A ˙ , L a 3 ' = L a 3 ρ ϕ ϵ 0
(036) L a ' = L a 1 ' + L a 2 ' + L a 3 ' = 1 2 ϕ 2 + ϕ A ˙ ρ ϕ ϵ 0
Jetzt ist es notwendig, wegzulassen L β 1 , Gleichung (025), der zweite Term ( ϕ A ˙ ) da es in erscheint L a 2 ' , siehe die zweite der obigen Gleichungen (035).

Also haben wir anstelle von (025)

(037) L β 1 ' 1 2 A ˙ 2
während L β 2 , L β 3 bleiben wie in den Gleichungen (027) und (029) unverändert
(038) L β 2 ' = L β 2 1 2 C 2 k = 1 k = 3 [ A X k A k A k 2 ] (039) L β 3 ' = L β 3 J A ϵ 0

Anstelle von (030)

(040) L β ' = L β 1 ' + L β 2 ' + L β 3 ' = 1 2 A ˙ 2 + 1 2 C 2 k = 1 k = 3 [ A X k A k A k 2 ] + J A ϵ 0
und schließlich für die neue Lagrange-Dichte, die wir anstelle von (031) haben

(041) L ' = L a ' + L β ' = 1 2 ϕ 2 + ϕ A ˙ ρ ϕ ϵ 0 + 1 2 A ˙ 2 + 1 2 C 2 k = 1 k = 3 [ A X k A k A k 2 ] + J A ϵ 0

Dichte L ' von (041) erhält man aus der Dichte L von (031), wenn wir den Term weglassen ( A ) ϕ . So L ' ist unabhängig von ϕ .

In den folgenden Gleichungen gruppiert die Klammer über den linken 3 Termen diesen Teil der Dichte L ' die wesentlich an der Erzeugung der elektromagnetischen Gleichung (006) aus der Lagrange-Gleichung bzgl. beteiligt ist ϕ , Gleichung (017), während die Klammer unter den rechten 4 Termen diesen Teil der Dichte gruppiert L ' das ist wesentlich an der Erzeugung der elektromagnetischen Gleichungen (008) aus den Lagrange-Gleichungen bzgl. beteiligt A 1 , A 2 , A 3 , Gleichung (022).

L ' = 1 2 ϕ 2 ρ ϕ ϵ 0 + ϕ A ˙ gegenüber  ϕ + 1 2 A ˙ 2 + 1 2 C 2 k = 1 k = 3 [ A X k A k A k 2 ] + J A ϵ 0

L ' = 1 2 ϕ 2 ρ ϕ ϵ 0 + ϕ A ˙ + 1 2 A ˙ 2 + 1 2 C 2 k = 1 k = 3 [ A X k A k A k 2 ] + J A ϵ 0 gegenüber  A

Beachten Sie den gemeinsamen Begriff ( ϕ A ˙ ) .

Neuordnung der Terme im Ausdruck (041) der Dichte L ' wir haben

L ' = 1 2 A ˙ 2 + 1 2 ϕ 2 + ϕ A ˙ 1 2 ϕ A T 2 1 2 C 2 k = 1 k = 3 [ A k 2 A X k A k ] × A 2 + 1 ϵ 0 ( ρ ϕ + J A )
(042)

das ist

(043) L ' = 1 2 | | ϕ A T | | 2 1 2 C 2 | | × A | | 2 + 1 ϵ 0 ( ρ ϕ + J A )
oder
(044) L ' = | | E | | 2 C 2 | | B | | 2 2 + 1 ϵ 0 ( ρ ϕ + J A )

Nun, wenn die Dichte L ' muss Energiedimensionen pro Volumeneinheit haben, die wir definieren L e M = ϵ 0 L ' So

(045) L e M = ϵ 0 | | E | | 2 C 2 | | B | | 2 2 ρ ϕ + J A
daran denken
(046a) E 2 = ϕ A T 2 = A ˙ 2 + ϕ 2 + 2 ( ϕ A ˙ ) (046b) B 2 = × A 2 = k = 1 k = 3 [ A k 2 A X k A k ]

Der Skalar ( | | E | | 2 C 2 | | B | | 2 ) ist eine der beiden Lorentz-Invarianten (2) des Körpers (die andere ist E B ) im Wesentlichen gleich einer konstanten Zeiten E μ v E μ v , Wo E μ v der Tensor des antisymmetrischen Feldes (2) .

Andererseits der Skalar ( ρ ϕ + J A ) ist im Wesentlichen das innere Produkt J μ A μ im Minkowski-Raum von zwei 4-Vektoren: die 4-Stromdichte J μ = ( C ρ , J ) und das 4-Potenzial A μ = ( ϕ / C , A ) , ebenfalls ein Lorentz-invarianter Skalar.

Also die Lagrange-Dichte L e M in Gleichung (045) ist Lorentz-invariant.


(1) Durch ein Trial-and-Error-Verfahren fand ich den Lagrange-Operator in einem schwierigeren und komplizierteren Fall: siehe meine Antwort als user82794 hier Erhalten Sie den Lagrange-Operator aus dem System der gekoppelten Gleichung

(2) In Anlehnung an W. Rindler in "Introduction to Special Relativity" Ed.1982 wird dieser Tensor in Gleichung (38.15) abgeleitet

(38.15) E μ v = [ 0 E 1 E 2 E 3 E 1 0 C B 3 C B 2 E 2 C B 3 0 C B 1 E 3 C B 2 C B 1 0 ] So E μ v = [ 0 E 1 E 2 E 3 E 1 0 C B 3 C B 2 E 2 C B 3 0 C B 1 E 3 C B 2 C B 1 0 ]
was durch die (Dualität) Ersetzungen E C B Und C B E Erträge
(39.05) B μ v = [ 0 C B 1 C B 2 C B 3 C B 1 0 E 3 E 2 C B 2 C E 3 0 E 1 C B 3 E 2 E 1 0 ] So B μ v = [ 0 C B 1 C B 2 C B 3 C B 1 0 E 3 E 2 C B 2 C E 3 0 E 1 C B 3 E 2 E 1 0 ]
Die beiden Invarianten von E μ v -an ihrer Entstehungsweise sofort als solche erkennbar - lassen sich wie folgt ausdrücken:
(39.06) X = 1 2 E μ v E μ v = 1 2 B μ v B μ v = C 2 | | B | | 2 | | E | | 2 (39.07) Y = 1 4 B μ v E μ v = C B E

Was ist der Unterschied zwischen „Trial and Error“ und „Raten“?
Schöner Latex
Ein Professor namens Bahman Zohuri vom Department of Electrical and Computer Engineering der University of New Mexico veröffentlichte am 30. Januar 2019 einen Artikel im PDF-Format mit dem Titel „Deriving the Lagrangian Density of an Electromagnetic Field“ mit einer genauen Kopie und Einfügung dieser Antwort hierin. Schade, dass in seinen Referenzen nicht einmal ein Wort über PhysicsStackExchange und meine Antwort fällt. Link : Ableitung der Lagrange-Dichte eines elektromagnetischen Feldes
... werfen Sie einen Blick in das genaue Kopieren und Einfügen von Bahman Zohuri ...
Es ist kaum zu glauben, dass ein Professor einer „renommierten“ Universität Ihre Welt plagiiert hat, ohne es auch nur zu erwähnen. Erstaunlicher ist, dass ein "renommierter" Verlag es veröffentlicht hat, gelinde gesagt ekelhaft.
@renormalizedQuanta : ... in der Tat, es ist schwer zu glauben ...
Diese Antwort ist großartig! (+++1)
@Frobenius, wegen dieses Kopierens und Einfügens denke ich, es wäre besser, wenn sich jemand an Springer wendet und dieses Plagiat meldet. Die renommierten Fachzeitschriften werfen den Professoren oft Plagiate vor. Ich bin mir ziemlich sicher, dass dies nicht das erste Mal ist, dass er das tut, und es wird nicht das letzte Mal sein.
In dem Moment, in dem ich dachte, dass Sie das veröffentlichen sollten, sah ich, dass ein Professor das (missbraucht) hatte!
@Deschele Schilder : ... was soll ich dazu sagen. Wie Sie sehen, poste ich diesen Kommentar am 4. Juni 19 und keine Antwort. Ich bin mir sicher, dass der Professor einer "renommierten" Universität und der "renommierte" Verlag davon wissen. Übrigens ist es immer eine Freude, Kommentare wie diesen hier zu sehen. Ein mathematisch unlogisches Argument bei der Herleitung der Hamilton-Gleichung in Goldstein .
Wie funktioniert in Gleichung (016) die μ 0 J gehe zu J k ϵ 0 ?
@moboDawn_φ : Willkommen bei PSE. Weil
C 2 ( 008 ) = ( 016 ) C 2 μ 0 J = J ϵ 0
Vielen Dank für diese tolle Antwort und Arbeit!
@abu_bua: Willkommen. Ich schlage vor, auch hier einen Blick darauf zu werfen. Warum ein komplexes Skalarfeld und sein komplexes Konjugat als zwei verschiedene Felder behandeln? für die Lagrange-Dichte der Schrödinger-Gleichung.
@Frobenius hast du dich beim Herausgeber/Verlag gemeldet?
@ziyuang: Danke für deine Aufmerksamkeit. Nein, ich habe mich nicht gemeldet. Ich glaube nicht, dass es sich lohnt, es zu tun.
@Frobenius Gute Antwort! Aber ein Kommentar! Die Maxwell-Gleichungen, die Sie geschrieben haben, sind keine Maxwell-Gleichungen im leeren Raum. Sie sind in Gegenwart von Ladungen und Strömen ( ρ , J 0 ).

Letztendlich muss die Begründung sein, dass (wie Sie sagten) so konstruiert werden muss, dass die Euler-Lagrange-Gleichungen Maxwell-Gleichungen sind. In gewisser Weise müssen Sie also den Lagrange erraten, der dies erzeugt, wie es hier zum Beispiel getan wird.

Sie können sich jedoch an der Tatsache orientieren, dass wir einen Lagrange-Operator für ein masseloses, nicht selbstwechselwirkendes Feld konstruieren müssen. Wir brauchen also eine Eich- und Lorentz-invariante Kombination des 4-Vektor-Potentials, die nur einen kinetischen Term hat (quadratisch in Ableitungen der Felder). Sie haben dann nicht viele Optionen abgesehen von F μ v F μ v . Der Quellterm ist dann bei Bedarf trivial hinzuzufügen.

Wie wäre es mit ϵ μ v σ τ F μ v F σ τ ?
Nun, ich sagte "nicht mehr viele Optionen" und tatsächlich würde die Kombination, die Sie aufschreiben, auch zu meinen Kriterien passen det ( F ) , aber man kann auch zuerst die einfachste Möglichkeit ausprobieren und das stellt sich als richtig heraus. Ich stelle fest, dass die von Ihnen angegebene Kombination ein Pseudoskalar ist. Kann sich jemand denken, ob es einen Grund gibt, warum das nicht erlaubt wäre?
@MistakeInk - ϵ μ v ρ σ F μ v F ρ σ ist ein guter Kandidat für den Lagragian, es ist nur eine totale Ableitung, so dass es das klassische EOM nicht beeinflusst und in der Störungstheorie verschwindet. Es hat jedoch immer noch einige Konsequenzen - siehe en.wikipedia.org/wiki/CP_violation#Strong_CP_problem . Wie für D e T ( F ) Ich glaube nicht, dass dieser Begriff renormierbar ist, da er gleich ist e T R Protokoll F die Sie um einen Hintergrundfeldwert erweitern und beliebig hohe Potenzen der Feldstärke erhalten könnten.
Sie erhalten jedoch Bedingungen des Formulars Protokoll T R ( k 2 + F μ v 2 ) bei der Berechnung der effektiven Einwirkung bei Vorhandensein eines Hintergrundfeld-Eichfeldes. Siehe Kapitel 16 von Peskin.
@DJBunk Da hast du Recht det ( F v μ ) ist nicht renormalisierbar, also würde es nicht funktionieren, aber es ist nur quartisch drin F und enthält keine willkürlich hohen Potenzen. Es ist auch nicht Lorentz-invariant.
@DJBunk Ein Wert ungleich Null ϵ F F Begriff wurde tatsächlich in realen experimentellen Systemen realisiert - 3D-topologische Isolatoren. Es wirkt sich nicht auf die Massenphysik aus, führt jedoch zu topologisch geschützten lückenlosen Kantenmoden an der Systemgrenze.

Ich bin mir fast zu 100% sicher, dass die Lagrange-Funktion eine Annahme der Theorie ist. Es kann nicht abgeleitet werden. Ich habe keine Referenzen für diese Behauptung. Ich weiß nur, dass aus jedem Kurs, den ich unterrichtet habe, und aus jedem Buch, das ich gelesen habe, die Lagrange-Funktion (vorausgesetzt, sie wird überhaupt verwendet) der Ausgangspunkt ist. Es ist in diesem Fall das „erste Prinzip“.

Danke - ich glaube, ich habe Lagrangeianer bisher nur von Mechanikern gesehen, wo sie von Natur aus die Form haben L = T v und damit das, was ich "ableitbar" nannte.
Ich sehe auch nicht, wie dieser Lagrangian "ableitbar" ist. Natürlich schreiben wir es so, dass uns die Euler-Lagrange-Gleichungen die klassischen Bewegungsgleichungen liefern. Als Ableitung würde ich das aber nicht bezeichnen. Es ist wirklich nur das Ersetzen einer Annahme durch eine andere.

Die Antwort finden Sie in dem Buch "Differential Geometry and Lie Groups for Physicists" von Marian Fecko.

In der geometrischen Sprache eine Wirkung eines Feldes F Ω P ( M ) auf einer n-dim Riemannschen Mannigfaltigkeit ( M , G ) sollte als 'inneres Produkt' verstanden werden

M F G F ,
Wo P < N , Und G ist der Hodge-Star-Operator, dh
G : Ω P ( M ) Ω N P ( M )
so dass die Wirkung diffeomorphismusinvariant ist und ihre Dichte ein Skalar ist.

Die Aktion eines freien Skalarfeldes hat beispielsweise folgende Form:

S [ ϕ ] = M D ϕ G D ϕ + M 2 M ϕ G ϕ = M | G | D N X { μ ϕ μ ϕ + M ϕ 2 2 } .
Wenn die 'Weltblatt'-Mannigfaltigkeit 1-dim ist, sind nur mögliche Felder 1-Formen. Man kann sich vorstellen, dass eine Aktion die folgende Form annimmt
R A μ D X μ D S D S
Wo A = A S D S = A μ D X μ D S D S ist eine 1-Form auf der Weltlinie, deren Hodge-Star-Dualfeld nicht definiert ist.

Sie können die Symmetrien von E & M verwenden, um zu zeigen, dass es im Wesentlichen nur einen vernünftigen Kandidaten gibt, der überprüft werden muss:

Die Wirkung, die wir suchen, sollte Lorentz-invariant, Eich-invariant, Paritäts- und Zeitumkehr-invariante und nicht mehr als zweite Ordnung in Ableitungen sein. Der einzige Kandidat ist [die Maxwell-Aktion]. [Srednicki QFT pg. 334.]